Is the following set a compact set?












2














Let $A$ be defined as



$$A:={fin C^1([0,1],mathbb{R}) : |f|_{C^1} leq 1}.$$



I have shown that the set is precompact. But is $A$ a complete set? Or an other question: Is $A$ a closed set?










share|cite|improve this question
























  • Which norm is $lVertcdotrVert_{C^1}$?
    – José Carlos Santos
    Nov 25 '18 at 9:55










  • How did You show $A$ is precompact?
    – Peter Melech
    Nov 25 '18 at 10:10
















2














Let $A$ be defined as



$$A:={fin C^1([0,1],mathbb{R}) : |f|_{C^1} leq 1}.$$



I have shown that the set is precompact. But is $A$ a complete set? Or an other question: Is $A$ a closed set?










share|cite|improve this question
























  • Which norm is $lVertcdotrVert_{C^1}$?
    – José Carlos Santos
    Nov 25 '18 at 9:55










  • How did You show $A$ is precompact?
    – Peter Melech
    Nov 25 '18 at 10:10














2












2








2







Let $A$ be defined as



$$A:={fin C^1([0,1],mathbb{R}) : |f|_{C^1} leq 1}.$$



I have shown that the set is precompact. But is $A$ a complete set? Or an other question: Is $A$ a closed set?










share|cite|improve this question















Let $A$ be defined as



$$A:={fin C^1([0,1],mathbb{R}) : |f|_{C^1} leq 1}.$$



I have shown that the set is precompact. But is $A$ a complete set? Or an other question: Is $A$ a closed set?







general-topology functional-analysis compactness complete-spaces locally-compact-groups






share|cite|improve this question















share|cite|improve this question













share|cite|improve this question




share|cite|improve this question








edited Nov 25 '18 at 17:37









Yiorgos S. Smyrlis

62.5k1383162




62.5k1383162










asked Nov 25 '18 at 9:44









MathCracky

445212




445212












  • Which norm is $lVertcdotrVert_{C^1}$?
    – José Carlos Santos
    Nov 25 '18 at 9:55










  • How did You show $A$ is precompact?
    – Peter Melech
    Nov 25 '18 at 10:10


















  • Which norm is $lVertcdotrVert_{C^1}$?
    – José Carlos Santos
    Nov 25 '18 at 9:55










  • How did You show $A$ is precompact?
    – Peter Melech
    Nov 25 '18 at 10:10
















Which norm is $lVertcdotrVert_{C^1}$?
– José Carlos Santos
Nov 25 '18 at 9:55




Which norm is $lVertcdotrVert_{C^1}$?
– José Carlos Santos
Nov 25 '18 at 9:55












How did You show $A$ is precompact?
– Peter Melech
Nov 25 '18 at 10:10




How did You show $A$ is precompact?
– Peter Melech
Nov 25 '18 at 10:10










2 Answers
2






active

oldest

votes


















4














The set is closed since it is the inverse image of $[0,1]$ with respect to the continuous map $lVertcdotrVert_{C^1}$.



But it cannot possibly be compact because the only normed spaces with compact closed unit balls are the finite-dimensional ones. And your space isn't.



I don't know which norm the norm $lVertcdotrVert_{C^1}$ is, but I suppose that it is such that $C^1bigl([0,1],mathbb Rbigr)$ is complete. If that's so, then $A$ is complete (since it is closed). Therefore, it cannot possibly be precompact, since compact $iff$ precompact and complete.






share|cite|improve this answer



















  • 2




    Good answer (+1). Just for general knowledge the norm $|cdot|_{C^1}$ is usually the sup norm + the sup norm of the derivative (i.e. $|f|_{C^1} = |f|_infty + |f'|_infty$). This is indeed a norm such that $C^1([0,1],mathbb{R})$ is complete.
    – Yanko
    Nov 25 '18 at 10:44










  • But the set $A$ is precompact in $C([0,1])$ with the sup norm by Arzela-Ascoli. Maybe that's what the op meant.
    – MaoWao
    Nov 25 '18 at 17:58










  • No, it is not precompact. If $f_n(x)=x^n$, $(f_n)_{ninmathbb N}$ has no convergent subsequence.
    – José Carlos Santos
    Nov 25 '18 at 18:04



















4














The answer is NO.



Consider the sequence
$$
f_n(x)=frac{1}{n+1}sin(nx),,,ninmathbb N.
$$

Then
$$
|,f_n|_{C^1}= max |,f_n|+ max |,f_n'|=frac{1}{n+1}+frac{n}{n+1}=1.
$$

If ${,f_n}$ possessed a converging subsequence $,{,f_{n_k}}$, in the $C^1-$sense, with limit $,f,,$ then $,{,f_{n_k}}$ would also converge to $f$ in the uniform sense. But, ${,f_{n_k}}$
converges uniformly to $fequiv 0$. Nevertheless, $,{,f_{n_k}}$ DOES NOT converge in the $C^1-$sense to $0$, since $,|,f_{n_k}|_{C^1}=1$.






share|cite|improve this answer





















    Your Answer





    StackExchange.ifUsing("editor", function () {
    return StackExchange.using("mathjaxEditing", function () {
    StackExchange.MarkdownEditor.creationCallbacks.add(function (editor, postfix) {
    StackExchange.mathjaxEditing.prepareWmdForMathJax(editor, postfix, [["$", "$"], ["\\(","\\)"]]);
    });
    });
    }, "mathjax-editing");

    StackExchange.ready(function() {
    var channelOptions = {
    tags: "".split(" "),
    id: "69"
    };
    initTagRenderer("".split(" "), "".split(" "), channelOptions);

    StackExchange.using("externalEditor", function() {
    // Have to fire editor after snippets, if snippets enabled
    if (StackExchange.settings.snippets.snippetsEnabled) {
    StackExchange.using("snippets", function() {
    createEditor();
    });
    }
    else {
    createEditor();
    }
    });

    function createEditor() {
    StackExchange.prepareEditor({
    heartbeatType: 'answer',
    autoActivateHeartbeat: false,
    convertImagesToLinks: true,
    noModals: true,
    showLowRepImageUploadWarning: true,
    reputationToPostImages: 10,
    bindNavPrevention: true,
    postfix: "",
    imageUploader: {
    brandingHtml: "Powered by u003ca class="icon-imgur-white" href="https://imgur.com/"u003eu003c/au003e",
    contentPolicyHtml: "User contributions licensed under u003ca href="https://creativecommons.org/licenses/by-sa/3.0/"u003ecc by-sa 3.0 with attribution requiredu003c/au003e u003ca href="https://stackoverflow.com/legal/content-policy"u003e(content policy)u003c/au003e",
    allowUrls: true
    },
    noCode: true, onDemand: true,
    discardSelector: ".discard-answer"
    ,immediatelyShowMarkdownHelp:true
    });


    }
    });














    draft saved

    draft discarded


















    StackExchange.ready(
    function () {
    StackExchange.openid.initPostLogin('.new-post-login', 'https%3a%2f%2fmath.stackexchange.com%2fquestions%2f3012634%2fis-the-following-set-a-compact-set%23new-answer', 'question_page');
    }
    );

    Post as a guest















    Required, but never shown

























    2 Answers
    2






    active

    oldest

    votes








    2 Answers
    2






    active

    oldest

    votes









    active

    oldest

    votes






    active

    oldest

    votes









    4














    The set is closed since it is the inverse image of $[0,1]$ with respect to the continuous map $lVertcdotrVert_{C^1}$.



    But it cannot possibly be compact because the only normed spaces with compact closed unit balls are the finite-dimensional ones. And your space isn't.



    I don't know which norm the norm $lVertcdotrVert_{C^1}$ is, but I suppose that it is such that $C^1bigl([0,1],mathbb Rbigr)$ is complete. If that's so, then $A$ is complete (since it is closed). Therefore, it cannot possibly be precompact, since compact $iff$ precompact and complete.






    share|cite|improve this answer



















    • 2




      Good answer (+1). Just for general knowledge the norm $|cdot|_{C^1}$ is usually the sup norm + the sup norm of the derivative (i.e. $|f|_{C^1} = |f|_infty + |f'|_infty$). This is indeed a norm such that $C^1([0,1],mathbb{R})$ is complete.
      – Yanko
      Nov 25 '18 at 10:44










    • But the set $A$ is precompact in $C([0,1])$ with the sup norm by Arzela-Ascoli. Maybe that's what the op meant.
      – MaoWao
      Nov 25 '18 at 17:58










    • No, it is not precompact. If $f_n(x)=x^n$, $(f_n)_{ninmathbb N}$ has no convergent subsequence.
      – José Carlos Santos
      Nov 25 '18 at 18:04
















    4














    The set is closed since it is the inverse image of $[0,1]$ with respect to the continuous map $lVertcdotrVert_{C^1}$.



    But it cannot possibly be compact because the only normed spaces with compact closed unit balls are the finite-dimensional ones. And your space isn't.



    I don't know which norm the norm $lVertcdotrVert_{C^1}$ is, but I suppose that it is such that $C^1bigl([0,1],mathbb Rbigr)$ is complete. If that's so, then $A$ is complete (since it is closed). Therefore, it cannot possibly be precompact, since compact $iff$ precompact and complete.






    share|cite|improve this answer



















    • 2




      Good answer (+1). Just for general knowledge the norm $|cdot|_{C^1}$ is usually the sup norm + the sup norm of the derivative (i.e. $|f|_{C^1} = |f|_infty + |f'|_infty$). This is indeed a norm such that $C^1([0,1],mathbb{R})$ is complete.
      – Yanko
      Nov 25 '18 at 10:44










    • But the set $A$ is precompact in $C([0,1])$ with the sup norm by Arzela-Ascoli. Maybe that's what the op meant.
      – MaoWao
      Nov 25 '18 at 17:58










    • No, it is not precompact. If $f_n(x)=x^n$, $(f_n)_{ninmathbb N}$ has no convergent subsequence.
      – José Carlos Santos
      Nov 25 '18 at 18:04














    4












    4








    4






    The set is closed since it is the inverse image of $[0,1]$ with respect to the continuous map $lVertcdotrVert_{C^1}$.



    But it cannot possibly be compact because the only normed spaces with compact closed unit balls are the finite-dimensional ones. And your space isn't.



    I don't know which norm the norm $lVertcdotrVert_{C^1}$ is, but I suppose that it is such that $C^1bigl([0,1],mathbb Rbigr)$ is complete. If that's so, then $A$ is complete (since it is closed). Therefore, it cannot possibly be precompact, since compact $iff$ precompact and complete.






    share|cite|improve this answer














    The set is closed since it is the inverse image of $[0,1]$ with respect to the continuous map $lVertcdotrVert_{C^1}$.



    But it cannot possibly be compact because the only normed spaces with compact closed unit balls are the finite-dimensional ones. And your space isn't.



    I don't know which norm the norm $lVertcdotrVert_{C^1}$ is, but I suppose that it is such that $C^1bigl([0,1],mathbb Rbigr)$ is complete. If that's so, then $A$ is complete (since it is closed). Therefore, it cannot possibly be precompact, since compact $iff$ precompact and complete.







    share|cite|improve this answer














    share|cite|improve this answer



    share|cite|improve this answer








    edited Nov 25 '18 at 10:03

























    answered Nov 25 '18 at 9:58









    José Carlos Santos

    150k22121221




    150k22121221








    • 2




      Good answer (+1). Just for general knowledge the norm $|cdot|_{C^1}$ is usually the sup norm + the sup norm of the derivative (i.e. $|f|_{C^1} = |f|_infty + |f'|_infty$). This is indeed a norm such that $C^1([0,1],mathbb{R})$ is complete.
      – Yanko
      Nov 25 '18 at 10:44










    • But the set $A$ is precompact in $C([0,1])$ with the sup norm by Arzela-Ascoli. Maybe that's what the op meant.
      – MaoWao
      Nov 25 '18 at 17:58










    • No, it is not precompact. If $f_n(x)=x^n$, $(f_n)_{ninmathbb N}$ has no convergent subsequence.
      – José Carlos Santos
      Nov 25 '18 at 18:04














    • 2




      Good answer (+1). Just for general knowledge the norm $|cdot|_{C^1}$ is usually the sup norm + the sup norm of the derivative (i.e. $|f|_{C^1} = |f|_infty + |f'|_infty$). This is indeed a norm such that $C^1([0,1],mathbb{R})$ is complete.
      – Yanko
      Nov 25 '18 at 10:44










    • But the set $A$ is precompact in $C([0,1])$ with the sup norm by Arzela-Ascoli. Maybe that's what the op meant.
      – MaoWao
      Nov 25 '18 at 17:58










    • No, it is not precompact. If $f_n(x)=x^n$, $(f_n)_{ninmathbb N}$ has no convergent subsequence.
      – José Carlos Santos
      Nov 25 '18 at 18:04








    2




    2




    Good answer (+1). Just for general knowledge the norm $|cdot|_{C^1}$ is usually the sup norm + the sup norm of the derivative (i.e. $|f|_{C^1} = |f|_infty + |f'|_infty$). This is indeed a norm such that $C^1([0,1],mathbb{R})$ is complete.
    – Yanko
    Nov 25 '18 at 10:44




    Good answer (+1). Just for general knowledge the norm $|cdot|_{C^1}$ is usually the sup norm + the sup norm of the derivative (i.e. $|f|_{C^1} = |f|_infty + |f'|_infty$). This is indeed a norm such that $C^1([0,1],mathbb{R})$ is complete.
    – Yanko
    Nov 25 '18 at 10:44












    But the set $A$ is precompact in $C([0,1])$ with the sup norm by Arzela-Ascoli. Maybe that's what the op meant.
    – MaoWao
    Nov 25 '18 at 17:58




    But the set $A$ is precompact in $C([0,1])$ with the sup norm by Arzela-Ascoli. Maybe that's what the op meant.
    – MaoWao
    Nov 25 '18 at 17:58












    No, it is not precompact. If $f_n(x)=x^n$, $(f_n)_{ninmathbb N}$ has no convergent subsequence.
    – José Carlos Santos
    Nov 25 '18 at 18:04




    No, it is not precompact. If $f_n(x)=x^n$, $(f_n)_{ninmathbb N}$ has no convergent subsequence.
    – José Carlos Santos
    Nov 25 '18 at 18:04











    4














    The answer is NO.



    Consider the sequence
    $$
    f_n(x)=frac{1}{n+1}sin(nx),,,ninmathbb N.
    $$

    Then
    $$
    |,f_n|_{C^1}= max |,f_n|+ max |,f_n'|=frac{1}{n+1}+frac{n}{n+1}=1.
    $$

    If ${,f_n}$ possessed a converging subsequence $,{,f_{n_k}}$, in the $C^1-$sense, with limit $,f,,$ then $,{,f_{n_k}}$ would also converge to $f$ in the uniform sense. But, ${,f_{n_k}}$
    converges uniformly to $fequiv 0$. Nevertheless, $,{,f_{n_k}}$ DOES NOT converge in the $C^1-$sense to $0$, since $,|,f_{n_k}|_{C^1}=1$.






    share|cite|improve this answer


























      4














      The answer is NO.



      Consider the sequence
      $$
      f_n(x)=frac{1}{n+1}sin(nx),,,ninmathbb N.
      $$

      Then
      $$
      |,f_n|_{C^1}= max |,f_n|+ max |,f_n'|=frac{1}{n+1}+frac{n}{n+1}=1.
      $$

      If ${,f_n}$ possessed a converging subsequence $,{,f_{n_k}}$, in the $C^1-$sense, with limit $,f,,$ then $,{,f_{n_k}}$ would also converge to $f$ in the uniform sense. But, ${,f_{n_k}}$
      converges uniformly to $fequiv 0$. Nevertheless, $,{,f_{n_k}}$ DOES NOT converge in the $C^1-$sense to $0$, since $,|,f_{n_k}|_{C^1}=1$.






      share|cite|improve this answer
























        4












        4








        4






        The answer is NO.



        Consider the sequence
        $$
        f_n(x)=frac{1}{n+1}sin(nx),,,ninmathbb N.
        $$

        Then
        $$
        |,f_n|_{C^1}= max |,f_n|+ max |,f_n'|=frac{1}{n+1}+frac{n}{n+1}=1.
        $$

        If ${,f_n}$ possessed a converging subsequence $,{,f_{n_k}}$, in the $C^1-$sense, with limit $,f,,$ then $,{,f_{n_k}}$ would also converge to $f$ in the uniform sense. But, ${,f_{n_k}}$
        converges uniformly to $fequiv 0$. Nevertheless, $,{,f_{n_k}}$ DOES NOT converge in the $C^1-$sense to $0$, since $,|,f_{n_k}|_{C^1}=1$.






        share|cite|improve this answer












        The answer is NO.



        Consider the sequence
        $$
        f_n(x)=frac{1}{n+1}sin(nx),,,ninmathbb N.
        $$

        Then
        $$
        |,f_n|_{C^1}= max |,f_n|+ max |,f_n'|=frac{1}{n+1}+frac{n}{n+1}=1.
        $$

        If ${,f_n}$ possessed a converging subsequence $,{,f_{n_k}}$, in the $C^1-$sense, with limit $,f,,$ then $,{,f_{n_k}}$ would also converge to $f$ in the uniform sense. But, ${,f_{n_k}}$
        converges uniformly to $fequiv 0$. Nevertheless, $,{,f_{n_k}}$ DOES NOT converge in the $C^1-$sense to $0$, since $,|,f_{n_k}|_{C^1}=1$.







        share|cite|improve this answer












        share|cite|improve this answer



        share|cite|improve this answer










        answered Nov 25 '18 at 10:55









        Yiorgos S. Smyrlis

        62.5k1383162




        62.5k1383162






























            draft saved

            draft discarded




















































            Thanks for contributing an answer to Mathematics Stack Exchange!


            • Please be sure to answer the question. Provide details and share your research!

            But avoid



            • Asking for help, clarification, or responding to other answers.

            • Making statements based on opinion; back them up with references or personal experience.


            Use MathJax to format equations. MathJax reference.


            To learn more, see our tips on writing great answers.





            Some of your past answers have not been well-received, and you're in danger of being blocked from answering.


            Please pay close attention to the following guidance:


            • Please be sure to answer the question. Provide details and share your research!

            But avoid



            • Asking for help, clarification, or responding to other answers.

            • Making statements based on opinion; back them up with references or personal experience.


            To learn more, see our tips on writing great answers.




            draft saved


            draft discarded














            StackExchange.ready(
            function () {
            StackExchange.openid.initPostLogin('.new-post-login', 'https%3a%2f%2fmath.stackexchange.com%2fquestions%2f3012634%2fis-the-following-set-a-compact-set%23new-answer', 'question_page');
            }
            );

            Post as a guest















            Required, but never shown





















































            Required, but never shown














            Required, but never shown












            Required, but never shown







            Required, but never shown

































            Required, but never shown














            Required, but never shown












            Required, but never shown







            Required, but never shown







            Popular posts from this blog

            Plaza Victoria

            In PowerPoint, is there a keyboard shortcut for bulleted / numbered list?

            How to put 3 figures in Latex with 2 figures side by side and 1 below these side by side images but in...